One step further.

w , x , y , z w, x, y, z are chosen uniformly at random from [ 0 , 1 ] . [0, 1].

Find the probability that w 2 + x 2 + y 2 + z 2 < 1 w^2+x^{2}+y^{2}+z^{2} < 1 to 3 decimal places.


The answer is 0.308.

This section requires Javascript.
You are seeing this because something didn't load right. We suggest you, (a) try refreshing the page, (b) enabling javascript if it is disabled on your browser and, finally, (c) loading the non-javascript version of this page . We're sorry about the hassle.

3 solutions

We will solve it using geometrical probability:

Our desired outcome lies in a sixteenth of a 4-dimensional sphere and all outcomes lie in a 4-dimensional cube so our probability is the hypervolume volume of the sphere over 16 times the hypervolume of the cube. Which is:

1 16 1 4 × π 2 2 1 4 = 0.308 \frac {1}{16} \frac {1^4 \times \frac {π^{2}}{2}}{1^{4}} = \boxed {0.308}

BTW The hypervolume of a 4D sphere with radius r r is given by r 4 × π 2 2 r^{4} \times \frac {π^{2}}{2} . I've derived this formula by realising that:

f n + 1 ( r ) = r r f n ( r 2 x 2 ) d x f_{n+1}(r) = \int_{-r}^r\ f_{n}(\sqrt{r^{2}-x^{2}}) dx

Where f n ( r ) f_{n}(r) is the hypervolume of an n-dimensional sphere with radius r r .

f 0 ( 1 ) = 1 f_{0}(1)=1 because 0-dimensional sphere has one point.

Just to let you guys know the probability after integrating comes out to be

Nicola Mignoni
Apr 6, 2018

It's possible to generalize this kind of probability problems using geometrical probability: if we have a 1 , a 2 , . . . a n a_1, a_2,...a_n uniformly chosen from [ 0 , b ] [0,b] , the probability of S = i = 1 n a i 2 < b 2 S=\displaystyle \sum_{i=1}^{n} {a_i}^2 < b^2 is the probability that point P = ( a 1 , . . . , a n ) P=(a_1,...,a_n) is inside the origin-centered n n -dimensions sphere with radius b b . We can express the value of this probability as

P ( P S ) = 1 b n 0 b 0 s 2 0 s 3 0 s n d a 1 d a 2 . . . d a n \displaystyle \mathbb{P}(P \subset S)= \frac{1}{b^n}\int_{0}^{b} \int_{0}^{s_2} \int_{0}^{s_3} \dots \int_{0}^{s_n} da_1 da_2...da_n

where

s i = b 2 k = 1 i 1 a k 2 \displaystyle s_i=\sqrt{b^2-\sum_{k=1}^{i-1} {a_k}^2} .

In our case n = 4 n=4 , b = 1 b=1 so

P ( P S ) = 0 b 0 1 a 1 2 0 1 a 1 2 a 2 2 0 1 a 1 2 a 2 2 a 3 2 d a 1 d a 2 d a 3 d a 4 0.308 \displaystyle \mathbb{P}(P \subset S)= \int_{0}^{b} \int_{0}^{\sqrt{1-{a_1}^2}} \int_{0}^{\sqrt{1-{a_1}^2-{a_2}^2}} \int_{0}^{\sqrt{1-{a_1}^2-{a_2}^2-{a_3}^2}} da_1 da_2 da_3 da_4 \approx \boxed{0.308}

0 pending reports

×

Problem Loading...

Note Loading...

Set Loading...